Difference between revisions of "2024 AMC 12A Problems/Problem 15"
(→Solution 4 (Reduction of power)) |
(→Solution 4 (Reduction of power)) |
||
Line 39: | Line 39: | ||
The motivation for this solution is the observation that <math>(p+c)(q+c)(r+c)</math> is easy to compute for any constant c, since <math>(p+c)(q+c)(r+c)=-f(-c)</math>, where <math>f</math> is the polynomial given in the problem. The idea is to transform the expression involving <math>p^2, q^2, r^2</math> into one involving <math>p, q, r</math>. | The motivation for this solution is the observation that <math>(p+c)(q+c)(r+c)</math> is easy to compute for any constant c, since <math>(p+c)(q+c)(r+c)=-f(-c)</math>, where <math>f</math> is the polynomial given in the problem. The idea is to transform the expression involving <math>p^2, q^2, r^2</math> into one involving <math>p, q, r</math>. | ||
− | Since <math>p^3+ | + | Since <math>p^3+2px^2-p+3=0</math>, we get <math>p^2=\frac{p-3}{p+2}</math>. Similarly <math>q^2=\frac{q-3}{q+2}</math> and <math>r^2=\frac{r-3}{r+2}</math>. |
Hence, | Hence, | ||
<cmath>(p^2 + 4)(q^2 + 4)(r^2 + 4)</cmath> | <cmath>(p^2 + 4)(q^2 + 4)(r^2 + 4)</cmath> | ||
− | <cmath>=\left(\frac{p-3}{p+ | + | <cmath>=\left(\frac{p-3}{p+2}+4\right)\left(\frac{q-3}{q+2}+4\right)\left(\frac{r-3}{r+2}+4\right)</cmath> |
− | <cmath>=\left(\frac{5p+ | + | <cmath>=\left(\frac{5p+5}{p+1}\right)\left(\frac{5q+5}{q+1}\right)\left(\frac{5r+5}{r+1}\right)</cmath> |
− | |||
− | |||
<cmath>=125</cmath> | <cmath>=125</cmath> | ||
− | + | Interestingly, we didn't even need to do any calculations as the terms just cancelled. Our answer is <math>\boxed{\textbf{(D) }125}</math>. | |
~tsun26 | ~tsun26 |
Revision as of 21:09, 8 November 2024
Contents
Problem
The roots of are and What is the value of
Solution 1
You can factor as .
For any polynomial , you can create a new polynomial , which will have roots that instead have the value subtracted.
Substituting and into for the first polynomial, gives you and as for both equations. Multiplying and together gives you .
-ev2028
~Latex by eevee9406
Solution 2
Let . Then .
We find that and , so .
~eevee9406
Solution 3
First, denote that Then we expand the expression ~lptoggled
Solution 4 (Reduction of power)
The motivation for this solution is the observation that is easy to compute for any constant c, since , where is the polynomial given in the problem. The idea is to transform the expression involving into one involving .
Since , we get . Similarly and .
Hence,
Interestingly, we didn't even need to do any calculations as the terms just cancelled. Our answer is .
~tsun26
See also
2024 AMC 12A (Problems • Answer Key • Resources) | |
Preceded by Problem 14 |
Followed by Problem 16 |
1 • 2 • 3 • 4 • 5 • 6 • 7 • 8 • 9 • 10 • 11 • 12 • 13 • 14 • 15 • 16 • 17 • 18 • 19 • 20 • 21 • 22 • 23 • 24 • 25 | |
All AMC 12 Problems and Solutions |
The problems on this page are copyrighted by the Mathematical Association of America's American Mathematics Competitions.